Question and Answers Forum

All Questions      Topic List

None Questions

Previous in All Question      Next in All Question      

Previous in None      Next in None      

Question Number 141525 by hgrocks last updated on 20/May/21

Let <x_n > be a sequence defined by  x_(n+1)  = (1/k)(x_n +(k/x_n )) ∀ n ∈ N  Show that <x_n > converges to (√(k/(k−1)))  x_1 >0 , k>1

$$\mathrm{Let}\:<\mathrm{x}_{\mathrm{n}} >\:\mathrm{be}\:\mathrm{a}\:\mathrm{sequence}\:\mathrm{defined}\:\mathrm{by} \\ $$ $$\mathrm{x}_{\mathrm{n}+\mathrm{1}} \:=\:\frac{\mathrm{1}}{\mathrm{k}}\left(\mathrm{x}_{\mathrm{n}} +\frac{\mathrm{k}}{\mathrm{x}_{\mathrm{n}} }\right)\:\forall\:\mathrm{n}\:\in\:\mathbb{N} \\ $$ $$\mathrm{Show}\:\mathrm{that}\:<\mathrm{x}_{\mathrm{n}} >\:\mathrm{converges}\:\mathrm{to}\:\sqrt{\frac{\mathrm{k}}{\mathrm{k}−\mathrm{1}}} \\ $$ $$\mathrm{x}_{\mathrm{1}} >\mathrm{0}\:,\:\mathrm{k}>\mathrm{1} \\ $$

Answered by mathmax by abdo last updated on 21/May/21

x_(n+1) =f(x_n ) with f(x)=(1/k)(x+(k/x))  f is continue on R^★   f^′ (x)=(1/k)(1−(k/x^2 ))  =(1/k)×((x^2 −k)/x^2 ) =(1/(kx^2 ))(x−(√k))(x+(√k))  we can limit the variation on ]0,+∞[ due to u_n >0 for all n  x              0                  (√k)                      +∞  f^′              ∣   ∣    −         0          +  f               ∣   ∣  dec       f((√k))     incr   +∞  due to continuity of f  the limit of u_(n ) is the fix point of f  f(x)=x  ⇒(1/k)(x+(k/x))=x ⇒x^2  +k=kx^2  ⇒(k−1)x^2  =k ⇒  but  x>0 ⇒x=(√(k/(k−1)))(   k>1)

$$\mathrm{x}_{\mathrm{n}+\mathrm{1}} =\mathrm{f}\left(\mathrm{x}_{\mathrm{n}} \right)\:\mathrm{with}\:\mathrm{f}\left(\mathrm{x}\right)=\frac{\mathrm{1}}{\mathrm{k}}\left(\mathrm{x}+\frac{\mathrm{k}}{\mathrm{x}}\right)\:\:\mathrm{f}\:\mathrm{is}\:\mathrm{continue}\:\mathrm{on}\:\mathrm{R}^{\bigstar} \\ $$ $$\mathrm{f}^{'} \left(\mathrm{x}\right)=\frac{\mathrm{1}}{\mathrm{k}}\left(\mathrm{1}−\frac{\mathrm{k}}{\mathrm{x}^{\mathrm{2}} }\right)\:\:=\frac{\mathrm{1}}{\mathrm{k}}×\frac{\mathrm{x}^{\mathrm{2}} −\mathrm{k}}{\mathrm{x}^{\mathrm{2}} }\:=\frac{\mathrm{1}}{\mathrm{kx}^{\mathrm{2}} }\left(\mathrm{x}−\sqrt{\mathrm{k}}\right)\left(\mathrm{x}+\sqrt{\mathrm{k}}\right) \\ $$ $$\left.\mathrm{we}\:\mathrm{can}\:\mathrm{limit}\:\mathrm{the}\:\mathrm{variation}\:\mathrm{on}\:\right]\mathrm{0},+\infty\left[\:\mathrm{due}\:\mathrm{to}\:\mathrm{u}_{\mathrm{n}} >\mathrm{0}\:\mathrm{for}\:\mathrm{all}\:\mathrm{n}\right. \\ $$ $$\mathrm{x}\:\:\:\:\:\:\:\:\:\:\:\:\:\:\mathrm{0}\:\:\:\:\:\:\:\:\:\:\:\:\:\:\:\:\:\:\sqrt{\mathrm{k}}\:\:\:\:\:\:\:\:\:\:\:\:\:\:\:\:\:\:\:\:\:\:+\infty \\ $$ $$\mathrm{f}^{'} \:\:\:\:\:\:\:\:\:\:\:\:\:\mid\:\:\:\mid\:\:\:\:−\:\:\:\:\:\:\:\:\:\mathrm{0}\:\:\:\:\:\:\:\:\:\:+ \\ $$ $$\mathrm{f}\:\:\:\:\:\:\:\:\:\:\:\:\:\:\:\mid\:\:\:\mid\:\:\mathrm{dec}\:\:\:\:\:\:\:\mathrm{f}\left(\sqrt{\mathrm{k}}\right)\:\:\:\:\:\mathrm{incr}\:\:\:+\infty \\ $$ $$\mathrm{due}\:\mathrm{to}\:\mathrm{continuity}\:\mathrm{of}\:\mathrm{f}\:\:\mathrm{the}\:\mathrm{limit}\:\mathrm{of}\:\mathrm{u}_{\mathrm{n}\:} \mathrm{is}\:\mathrm{the}\:\mathrm{fix}\:\mathrm{point}\:\mathrm{of}\:\mathrm{f} \\ $$ $$\mathrm{f}\left(\mathrm{x}\right)=\mathrm{x}\:\:\Rightarrow\frac{\mathrm{1}}{\mathrm{k}}\left(\mathrm{x}+\frac{\mathrm{k}}{\mathrm{x}}\right)=\mathrm{x}\:\Rightarrow\mathrm{x}^{\mathrm{2}} \:+\mathrm{k}=\mathrm{kx}^{\mathrm{2}} \:\Rightarrow\left(\mathrm{k}−\mathrm{1}\right)\mathrm{x}^{\mathrm{2}} \:=\mathrm{k}\:\Rightarrow \\ $$ $$\mathrm{but}\:\:\mathrm{x}>\mathrm{0}\:\Rightarrow\mathrm{x}=\sqrt{\frac{\mathrm{k}}{\mathrm{k}−\mathrm{1}}}\left(\:\:\:\mathrm{k}>\mathrm{1}\right) \\ $$

Commented byhgrocks last updated on 21/May/21

Thanks a lot .★★

$$\mathrm{Thanks}\:\mathrm{a}\:\mathrm{lot}\:.\bigstar\bigstar \\ $$ $$ \\ $$

Terms of Service

Privacy Policy

Contact: info@tinkutara.com